LSAT and Law School Admissions Forum

Get expert LSAT preparation and law school admissions advice from PowerScore Test Preparation.

 Jon Denning
PowerScore Staff
  • PowerScore Staff
  • Posts: 904
  • Joined: Apr 11, 2011
|
#37271
Another List question to start, meaning three of the four games (all but game 2) began with a question like this. That's three easy points if you follow the simple guideline of using the rules to eliminate the four wrong answers.

So let's begin doing that, but starting with what's likely to be the easiest rule to spot. My vote is probably "no G in S."

And immediately we see that answer choice (B) is out.

Next I suspect that a violation of the FH Not Block (no F and H together) should be highly visible...and sure enough answer choice (E) is gone.

What about the G at M, H at T rules? Answer choice (D) breaks it: G is at M, and T has F and I.

And lastly let's compare (A) and (C) for the I goes twice rule: answer choice (A) has a single I, so it's out and (C) is the right answer.

Why did I save the two Is rule for last? It requires me to potentially count through everything, and doing that for only two answers is far easier than doing it for all five. Be smart as you apply the rules; start with most obvious or visible and work your way down. It may only save a few seconds, but every second counts.

Get the most out of your LSAT Prep Plus subscription.

Analyze and track your performance with our Testing and Analytics Package.